Mobius Transformations, quick question concepts.

Click For Summary
A Mobius transformation is defined as f(z) = (az + b) / (cz + d), with the condition ad - bc ≠ 0 being crucial. This condition ensures that the transformation is invertible; if ad - bc = 0, the function becomes constant, mapping all points to a single value. The discussion highlights confusion over why the case ad - bc = 0 is described as "leaving all points unchanged," as it does not preserve the original points but rather collapses them to a constant. The distinction is made that the identity map is not achieved under these conditions, as it does not satisfy the requirement for invertibility. Understanding this condition is essential for grasping the nature of Mobius transformations.
binbagsss
Messages
1,291
Reaction score
12
So a mobius transformation is defined as \frac{az+b}{cz+d}=f(z).
Where ad-bc≠0.


My question is just deriving this condition ad-bc≠0.

I understand that the condition describes the case were the mapping leaves all points unchanged. This is described as undefined in some textbooks...(why is this, isn't it then just an identity map?)

But not by setting z=f(z).

I can't see why this condition would not equally be desribed by z=f(z)?

Many Thanks for any assistance.
 
Physics news on Phys.org
If ad-bc=0 then f(z) = b/d which is a constant.
Therefore, the equation y=f(z) can be inverted with respect to z only if ad-bc is not zero.
 
But I can't see why this condition would not equally be desribed by z=f(z)?
 
binbagsss said:
But I can't see why this condition would not equally be desribed by z=f(z)?

I guess I don't see why anyone would describe that mapping where ad-bc=0 as 'leaving all points unchanged'. It doesn't. It maps everything into a constant.
 
Question: A clock's minute hand has length 4 and its hour hand has length 3. What is the distance between the tips at the moment when it is increasing most rapidly?(Putnam Exam Question) Answer: Making assumption that both the hands moves at constant angular velocities, the answer is ## \sqrt{7} .## But don't you think this assumption is somewhat doubtful and wrong?

Similar threads

  • · Replies 2 ·
Replies
2
Views
2K
  • · Replies 4 ·
Replies
4
Views
2K
  • · Replies 2 ·
Replies
2
Views
2K
  • · Replies 2 ·
Replies
2
Views
2K
  • · Replies 1 ·
Replies
1
Views
2K
  • · Replies 23 ·
Replies
23
Views
2K
Replies
4
Views
4K
Replies
10
Views
8K
Replies
4
Views
2K
Replies
2
Views
2K